Đến nội dung

Hình ảnh

Tuần 1 tháng 12/2016 : Bài toán vuông góc trên cấu hình tiếp xúc

hình học

  • Please log in to reply
Chủ đề này có 6 trả lời

#1
baopbc

baopbc

    Himura Kenshin

  • Thành viên nổi bật 2016
  • 410 Bài viết

Như vậy thầy Hùng đã đưa ra lời giải bài cũ trong tuần 1 tháng 12 và kèm theo đó là bài toán mới, xin trích dẫn lại bài toán mới,

 

Cho tam giác $ABC$ nhọn có đường tròn nội tiếp $(I)$ tiếp xúc $BC,CA,AB$ tại $D,E,F$. Đường tròn qua $B,C$ tiếp xúc $(I)$ tại $P$. $K$ là hình chiếu của $D$ lên $EF$. $PK$ cắt $(I)$ tại $L$. Chứng minh rằng $DL\perp AI$.


Bài viết đã được chỉnh sửa nội dung bởi baopbc: 04-12-2016 - 18:03


#2
baopbc

baopbc

    Himura Kenshin

  • Thành viên nổi bật 2016
  • 410 Bài viết

Post 366.PNG

Giải. Gọi $M,N$ lần lượt là trung điểm $DE,DF$. $Q$ là giao điểm của $MN$ với $BC$.

Theo hệ thức lượng trong tam giác vuông, $\overline{IN}\cdot \overline{IB}=ID^2=\overline{IM}\cdot \overline{IC}$ nên tứ giác $BNMC$ nội tiếp.

$\implies \overline{QN}\cdot \overline{QM}=\overline{QB}\cdot \overline{QC}$. $\qquad (1)$

Do đường tròn $(DMN)$ tiếp xúc $(I)$ nên $QD$ là tiếp tuyến của $(DMN)$ $\implies QD^2=\overline{QN}\cdot \overline{QM}$ $\qquad (2)$

Từ $(1)$ và $(2)$ ta suy ra $QD^2=\overline{QB}\cdot \overline{QC}\implies \mathcal{P}_{Q/(I)}=\mathcal{P}_{Q/(O)}$ hay $Q$ thuộc trục đẳng phương của $(O)$ và $(I)$.

Mặt khác theo định lí về tâm đẳng phương, tiếp tuyến tại $P$ của $(I)$ đi qua $Q$. $\qquad (3)$

Gọi $R,S$ là giao điểm của $MN$ với $(I)$, từ $(3)$ thì $DRPS$ là tứ giác điều hòa. $\qquad (4)$

Đường thẳng qua $D$ vuông góc với $DK$ cắt $(I)$ tại $L'$. $T,U$ lần lượt là hình chiếu của $L',I$ lên $EF$. $V$ là trung điểm $KL'$.

Do $EFDL'$ là hình thang cân nên $T$ đối xứng $T$ qua $U$ $\implies IV\parallel L'T$ hay $IV\perp MN\implies V$ là trung điểm $RS$.

Từ đó $L'(DVRS)=-1$. Mặt khác từ $(4)$ thì $L'(DPRS)=-1$ nên $L',K,P$ thẳng hàng. Do đó $L\equiv L'$.

$\implies DL\perp DK\implies DL\perp AI$.

______________________

Bài này là biến thể của bài toán IMO Shortlist 2011 G4 quen thuộc, lời giải trên của em cũng dựa trên bài toán này.


Bài viết đã được chỉnh sửa nội dung bởi baopbc: 07-12-2016 - 10:58


#3
Ngockhanh99k48

Ngockhanh99k48

    Trung sĩ

  • Thành viên
  • 127 Bài viết
Một lời giải khác:
Tiếp tuyến tại $P$ cắt $BC$ tại $X$. Gọi $Y$ là đối xứng của $D$ qua $X$. Ta có $XD^2=XP^2=XY^2=\overline{XB}.\overline{XC}$. Do đó $(YDBC)=-1$ và $Y$ thuộc $EF$. Từ đó ta có $Y, P, K, D$ cùng thuộc đường tròn đường kính $YD$. Ta có $\widehat{PLD}+\widehat{DKL}=\widehat{PDY}+\widehat{PYD}=90^{\circ}$ hay là $DL \perp DK$. Do $DK \perp EF$, $EF \perp AI$ nên $DK \parallel AI$. Ta có đpcm.

#4
Ngockhanh99k48

Ngockhanh99k48

    Trung sĩ

  • Thành viên
  • 127 Bài viết
Bài toán tổng quát:
Cho tam giác $\triangle ABC$ với $P, Q$ là hai điểm liên hợp đẳng giác nằm trên phân giác góc $\widehat{BAC}$. Gọi $\triangle P_aP_bP_c$ và $\triangle Q_aQ_bQ_c$ là tam giác pedal ứng với hai điểm $P, Q$ của $\triangle ABC$. Gọi $X, Y$ là hình chiếu của $P_a, Q_a$ trên $Q_bQ_c, P_bP_c$. Kẻ $P_aP_a'$ và $Q_aQ_a'$ vuông góc với $PQ$. $P_a'Y$ và $Q_a'X$ cắt $(P_aP_bP_c)$ tại điểm $M, N$. Khi đó $B, M, N, C$ đồng viên.

Bài viết đã được chỉnh sửa nội dung bởi Ngockhanh99k48: 04-12-2016 - 23:11


#5
ecchi123

ecchi123

    Trung sĩ

  • Điều hành viên OLYMPIC
  • 177 Bài viết

Nhận thấy $(PKD)$ là đường tròn Apolo dựng trên đoạn $BC$ tỉ số$\frac{DB}{BC}$ , $EF$ cắt $(PKD)$ tại $M$ có$\widehat{DLC}=\widehat{DPL}=\widehat{KMD}$ suy ra $DL$ ss $EF$ tức vg góc $AI$


~O)  ~O)  ~O)


#6
quanghung86

quanghung86

    Thiếu úy

  • Điều hành viên
  • 632 Bài viết

Mở rộng hơn nữa như sau, chỉ cần $P,Q$ đẳng giác.

 

Figure4160.png

 

Cho tam giác $ABC$ với $P,Q$ là hai điểm đẳng giác. $P_a,P_b,P_c$ là hình chiếu của $P$ trên $BC,CA,AB$. $Q_a,Q_b,Q_c$ là hình chiếu của $Q$ trên $BC,CA,AB$. Thì sáu điểm $P_a,P_b,P_c,Q_a,Q_b,Q_c$ nằm trên đường tròn $(K)$. $Q_k$ là hình chiếu của $P_a$ trên $Q_bQ_c$. $P_k$ là hình chiếu của $Q_a$ trên $P_bP_c$. $P_l$ là đối xứng của $P_a$ qua trung trực $P_bP_c$. $Q_l$ là đối xứng của $Q_a$ qua trung trực $Q_bQ_c$. $P_kP_l$ cắt $(K)$ tại $P^*$. $Q_kQ_l$ cắt $(K)$ tại $Q^*$. Chứng minh rằng bốn điểm $B,C,P^*,Q^*$ cùng thuộc một đường tròn.


Bài viết đã được chỉnh sửa nội dung bởi quanghung86: 08-12-2016 - 17:56


#7
quanghung86

quanghung86

    Thiếu úy

  • Điều hành viên
  • 632 Bài viết

Cám ơn Bảo đã chỉ ra bài IMO Shortlist 2014 G4. Dựa trên đó, ta có thể đề xuất bài toán sau

 

Cho tam giác $ABC$ nội tiếp $(O)$ với các tiếp tuyến qua $B,C$ của $(O)$ cắt tiếp tuyến qua $A$ tại $Q,R$. $AH$ là đường cao của tam giác. $X$ đối xứng $A$ qua trung trực $BC$. $XH$ cắt $(O)$ tại $P$ khác $X$. Chứng minh rằng đường tròn $(PQR)$ tiếp xúc $(O)$.







Được gắn nhãn với một hoặc nhiều trong số những từ khóa sau: hình học

0 người đang xem chủ đề

0 thành viên, 0 khách, 0 thành viên ẩn danh